Is it possible to catch the Thief ? || Quick and Tricky Puzzle

Sdílet
Vložit
  • čas přidán 11. 10. 2019
  • Guys !! Don't forget to press the bell icon to get notified for every new upload.
    #Logic puzzle
    #Interview riddles
    #interview puzzles
    The task is to catch the thief by following the game rules.
    1 - A person can only move along one edge at a time.
    2 - They will make moves one by one until the police offer catches the thief.
    They can keep on making moves one by one in any direction.
    3 - If the police has to make the first move,
    Is it ever possible to catch the thief ?
    You are most welcome to share puzzle, math problems or any topics for upcoming videos.
    Gmail : logicreloaded@gmail.com
    Facebook(message) : / mohammmedammar
    I got this question from a CZcams video called ""Is it possible? Simple questions, not so simple solutions"" from a channel called MajorPrep. The solution shown in that video is a little bit different than what I explained.

Komentáře • 501

  • @salmanaljazari1560
    @salmanaljazari1560 Před 4 lety +432

    Me after seeing the thief can just cycle around the first square.
    "No the thief cannot be caught"

  • @GamePerFrame
    @GamePerFrame Před 4 lety +359

    As a Chess player, all I can say is this:
    - Opposition
    - Triangulation

    • @LOGICALLYYOURS
      @LOGICALLYYOURS  Před 4 lety +17

      That makes 100% sense

    • @ananyaj7914
      @ananyaj7914 Před 4 lety +9

      Wow even I too National level Chess player!

    • @LOGICALLYYOURS
      @LOGICALLYYOURS  Před 4 lety +17

      @@ananyaj7914 good to see that we have chess players here... You must be 2300 rated or higher!

    • @VirgiAhmad
      @VirgiAhmad Před 4 lety +34

      U forgot something
      -Zugzwang

    • @GamePerFrame
      @GamePerFrame Před 4 lety +1

      @@VirgiAhmad +1

  • @sykomantis
    @sykomantis Před 4 lety +252

    Here's my problem with the solution: what is stopping the thief from using the diagonal to swing the parity back in his favor?

    • @TlalocTemporal
      @TlalocTemporal Před 4 lety +48

      He can't reach it if the cop plays with anything but random moves.

    • @etro2649
      @etro2649 Před 4 lety +71

      The fact that the cop is closer to the diagonal means that the cop would get there first. While the cop is by the diagonal, the thief cant go near so it must go in the opposite direction of the diagonal. Now the cop can chase the thief back in the opposite direction and the thief cannot turn around.

    • @Survivor235
      @Survivor235 Před 4 lety +1

      Simple the cop will block thief because he is the first who can reach the diagonal line Period

    • @YashRaj-wn6zi
      @YashRaj-wn6zi Před 3 lety

      If thief is in A or B and cop on C , he is caught

    • @Bhaveshh1
      @Bhaveshh1 Před 3 lety +2

      And why thief must move everytime?? He should move only when the parity is even,its said they keep on making moves and my move would be too not move at all.

  • @anandbhalerao8130
    @anandbhalerao8130 Před 4 lety +232

    Dude i am pretty sure the Inventor forgot to take his phone in the bathroom and had to timepass by counting tiles

  • @karljay7473
    @karljay7473 Před 4 lety +71

    It should be noted that a move is required, meaning that both the police and robber must move each time.

  • @pradeepbwr
    @pradeepbwr Před 4 lety +22

    3 chess strategies are used
    i) opposition
    ii) triangulation
    iii) zugzuang

    • @LOGICALLYYOURS
      @LOGICALLYYOURS  Před 4 lety +1

      I agree with you... except 'Zugzuang' in this very situation.. I think 'Zugzuang' will be applicable only if we switch the positions of thief and police. But yes, Zugzuang will play an important role in this type of puzzles.

    • @dlevi67
      @dlevi67 Před 3 lety +2

      @@LOGICALLYYOURS It's a German word, and it's spelled Zugzwang, meaning 'forced to move'. It is in fact _the_ key to the solution here; if the thief is forced to make a move then the cop can catch him; otherwise the thief can always re-establish parity by not moving.

    • @PrimoStracciatella
      @PrimoStracciatella Před 3 lety

      @@dlevi67 You're right, I was wondering what "Zugzuang" means, it looks Chinese but sounds German.

  • @stevemaurer8120
    @stevemaurer8120 Před 3 lety +12

    The puzzle doesn't mention the most critical part: the thief HAS to move each turn. It's not optional.

  • @randysalber4960
    @randysalber4960 Před 4 lety +37

    I considered the chess idea that if the beginning scenario were the same, but i was the thief's obligation to move, he would be caught. Triangulation!

    • @leif1075
      @leif1075 Před 4 lety +1

      There's a mistake in the video though, at 3:06 why does he have thief move towards the cop instead of away, then there would have been a distance of 3 there but bthevthief could then get away..he's assuming the thief is dumb..

    • @roppis9258
      @roppis9258 Před 4 lety +4

      @@leif1075 The thief can't get away. It doesn't matter if he is 2 segments away or 4 segments away, because either way the police can catch him when the amount of segments between them becomes odd

    • @leif1075
      @leif1075 Před 4 lety

      @@roppis9258 yea but the segments won't necessarily be am odd distance if the thief is careful...that's my point...it's not necessarily going to happen

    • @roppis9258
      @roppis9258 Před 4 lety

      @@leif1075 Yes they are. After each turn the the thief can only ever make a move which moves him 1 away or closer to the police. That would always make an odd amount turn into even or an even amount turn into odd. The same is true for the police, expect for the diagonal segment which doesn't change his distance from the thief from even to odd or odd to even, but instead keeps it the same. Since they started with an even distance between them and the police goes first, this forces the thief to make a move which makes the distance between them swap from even to odd.

    • @leif1075
      @leif1075 Před 4 lety

      @@roppis9258 yea but they could,be locked,in a perpetual,back,and forth so the thief can get away in those,cases he won't always be caught

  • @ca352256
    @ca352256 Před 4 lety +7

    For all of you that don't understand this. This whole scenario is based on what the cop does. Who cares what the thief does because when the cop gets to the bottom right corner first (which he will based on the positions in the picture), the cop will then have a choice. He could move 1 segment (by going up or left) or move 2 segments (by using the diagonal). This means regardless of what the thief chooses, the cop can always remain an odd number of segments behind since the scenario is in a confined space with both parties being forced to move 1 segment per turn.

  • @elvinzhuo3007
    @elvinzhuo3007 Před 4 lety +14

    The police always carries a gun

    • @OneWeirdDude
      @OneWeirdDude Před 4 lety +1

      Pretend this is either England or Commiefornia.

  • @kslaxmisarathy4859
    @kslaxmisarathy4859 Před 4 lety +64

    The thief can just move between two points making it impossible for the police to catch

    • @rshanthikumar6224
      @rshanthikumar6224 Před 4 lety

      Yes the theif can do that.. But here he said is very confusing

    • @somerandommember
      @somerandommember Před 4 lety +22

      Kslaxmi Sarathy only if the police man chases after and stays even moves away. If the police man instead leaves for the diagonal line then he can come back and trap the thief since he’ll then be odd moves away.

    • @cakatikaggarwal2618
      @cakatikaggarwal2618 Před 4 lety +15

      "You don't listen .. do you" - The Joker

    • @brandonstack3672
      @brandonstack3672 Před 4 lety +4

      It only makes sense because they cannot stay in the same place. It's stupid.

    • @leif1075
      @leif1075 Před 4 lety +1

      @@somerandommember no,but if they can only,move,one edge at a time as per the rules, they could literally be locked in a perpetual cycle with the thief never being caught..

  • @LokiDaFerret
    @LokiDaFerret Před 3 lety +1

    I like your videos but in this case I have to call foul. The rules state that you must move along an edge. A diagonal is not an edge. Perhaps the rules could be more clearly stated that they must move from one junction point to another.

  • @KS-iq1zj
    @KS-iq1zj Před 4 lety +14

    Wish this who happen in real life situation then there would be no robbery.

  • @allblue801
    @allblue801 Před 4 lety +11

    This works a lot like chess, trying to checmate the thief. Almost too easy

    • @I_am_Itay
      @I_am_Itay Před 4 lety +2

      Exactly, Im playing professional chess so I solved that even before he told the rules.

    • @LOGICALLYYOURS
      @LOGICALLYYOURS  Před 4 lety +3

      I agree! It's analogous to chess end game with 2 Kings and a Pawn.

    • @I_am_Itay
      @I_am_Itay Před 4 lety

      @@LOGICALLYYOURS Exactly, just like opsition. Do you play chess?

    • @LOGICALLYYOURS
      @LOGICALLYYOURS  Před 4 lety +4

      @@I_am_Itay Yes, I do... not very professionally, but I play for brain storming. My lichess classic rating is around 2000, and puzzle rating has touched 2200. If you are on lichess, pls share your ID, or send me an invite @ammar97 .

    • @jiogcyihsugyiocjfdoivhphvw6821
      @jiogcyihsugyiocjfdoivhphvw6821 Před rokem

      wtf is checmate

  • @muratakdanali5361
    @muratakdanali5361 Před 4 lety +5

    Absolutely incredible !! İm so happy to discover ur channel keep the videos greetings from Turkey

  • @ChangerYK
    @ChangerYK Před 4 lety +17

    2:57 “thief can be caugth”

  • @aradhyamn6950
    @aradhyamn6950 Před 4 lety +3

    I love the ODD and EVEN concepts application in some of the puzzles..
    Like FOX in 5 holes.. Etc.
    Awesome sir ❤

    • @CUBINGGURU
      @CUBINGGURU Před 4 lety

      That question was solvable this is not

  • @kidsgameslearning
    @kidsgameslearning Před 2 lety +1

    Very nice puzzle and solving knowledge. Thief can use the similar diagonal to negate the police action to escape from the police.

  • @kavishramchandani3282
    @kavishramchandani3282 Před 4 lety +1

    3:14......and undertaker takes over!!

  • @kmayank8287
    @kmayank8287 Před 3 lety +1

    At 3:13 if the thief had moved to front instead of moving backwards the distance would have been 2 again. So the answer is he cannot be caught until he surrenders.

    • @iobudgerigar9133
      @iobudgerigar9133 Před 3 lety +1

      Then the police could move across the diagonal and corner him.

    • @svilenacarapica4491
      @svilenacarapica4491 Před 2 lety

      @@iobudgerigar9133 How so ? In the video thief is going right, right, right, left. He couldve gone right right left and so on.

    • @bindaldesai
      @bindaldesai Před 2 lety

      then it will be easier for police to catch him. earlier than one approach shown in video

  • @Talinthis
    @Talinthis Před 4 lety +6

    i am amazed at all the peoples comments that say this does not work

    • @You12783
      @You12783 Před 4 lety

      Yeah, if someone would've asked me this in interview I would've said as long as position of cop and crook aren't mentioned let them be situated in a single edged right next to one another... So just based on this assumption you could see that it's definitely possible. 😅

    • @jiogcyihsugyiocjfdoivhphvw6821
      @jiogcyihsugyiocjfdoivhphvw6821 Před rokem

      its bc they forgot their brains

  • @Aman-ns1yf
    @Aman-ns1yf Před 3 lety +6

    Why will the thief ever move away from the 1st square itself? 😂

    • @luunhattruong2384
      @luunhattruong2384 Před 3 lety

      after the police cross the diagonal line, the thief is guaranteed to be caught no matter where he stands

    • @dlevi67
      @dlevi67 Před 3 lety

      @@luunhattruong2384 Not true. If the thief does not have to move, he can re-establish parity by not moving for one turn.

    • @luunhattruong2384
      @luunhattruong2384 Před 3 lety

      @@dlevi67 well, that's against the rule and similar to chess, you cannot skip your move

    • @dlevi67
      @dlevi67 Před 3 lety

      @@luunhattruong2384 Have you actually read the original post that you 'answered' to? (And FWIW, Ammar did NOT say anywhere that both cop and thief have to move - though he should have).

    • @luunhattruong2384
      @luunhattruong2384 Před 3 lety

      @@dlevi67idk, I thought the guy meant the thief was harder to caught if he just circle around 4 vertices of the first square.

  • @jeremiebenoit1
    @jeremiebenoit1 Před 4 lety +4

    Nice puzzle, reminds me of triangulation and opposition in chess

  • @tamirerez2547
    @tamirerez2547 Před 4 lety +3

    As always great graphic explanation.
    And a very nice riddle.

  • @argsahoo
    @argsahoo Před 4 lety +4

    Really tricky puzzle, it's easy but at the first try it seems too hard

  • @michaelpurtell4741
    @michaelpurtell4741 Před 4 lety +4

    Change parity of moves via the diagonal

  • @SuperJunkyMonky
    @SuperJunkyMonky Před 4 lety +23

    If the thief was actually intelligent then no

  • @aradhyamn6950
    @aradhyamn6950 Před 4 lety +6

    Hi... Sir..
    U make one video regarding following question..
    " Two days ago , a boy was 8 year old.
    After One year he is 11 year old."
    How is that possible??

    • @praveengovindarajan4257
      @praveengovindarajan4257 Před 4 lety

      If a boy has a birthday on 1st Jan it can be solved

    • @LOGICALLYYOURS
      @LOGICALLYYOURS  Před 4 lety +1

      Aradhya... I just noted your question, and it will be shared very soon. Thanks for sharing it... and yes I solved it :)

    • @karanvirsingh2830
      @karanvirsingh2830 Před 4 lety +2

      Is his name 8 year old🤐😂😂😂

    • @nathancarlisle2094
      @nathancarlisle2094 Před 4 lety

      I honestly can't think how that would work. I can think of lots of scenarios where an 8 year old can be 10 one year later but I can't find a solution for 11.

    • @vladislav_sidorenko
      @vladislav_sidorenko Před 4 lety

      @@nathancarlisle2094 I assume that "After One year he is 11 year old" means that the boy will be 11 years old on the next year, not after one year specifically. Then, the boy's birthday will be on the 31st of December, and today is the 1st of January.

  • @osrengo6643
    @osrengo6643 Před 4 lety +5

    Easy for every decent Chess Player

    • @ritvikkaran967
      @ritvikkaran967 Před 4 lety

      can you find out the solution for this loophole of metro
      czcams.com/video/DCo8o4q_GNA/video.html

    • @Majty.
      @Majty. Před 3 lety

      Easy fór anyone with IQ value of atleast 2 digits

  • @yatminghui8955
    @yatminghui8955 Před rokem +1

    2:56: With *Odd number* off segments, the theif can be *caugth*.
    what (look at the last word)

  • @kaukabesrar401
    @kaukabesrar401 Před 3 lety +2

    How will you catch the thief if he just moves up and down from the initial position???

  • @chillmode1881
    @chillmode1881 Před 2 lety

    The thief can easily counter the move. Try to look for it. Sympathize for the thief as well haha.
    Hint: if the thief tries his best to stay in the second upper dot, he can counter the police's wise move by moving right. There is more, look for it.
    This puzzle is imposibble.

  • @arunacharya3128
    @arunacharya3128 Před 4 lety +5

    I also solve this puzzle🤘🤘

  • @tmc9223
    @tmc9223 Před 3 lety +1

    After trying many possibilities once police go through the diagonal then thief must be caught....that's awesome......😁😁😁😁 really I tried many options..... every time thief got caught after crossing the diagonal

    • @sybro9786
      @sybro9786 Před 3 lety

      With perfect movement from the thief it will never cross the diagonal itself

  • @bitlaps
    @bitlaps Před 4 lety +6

    When the thief won't see the police from is first position
    what is the need to move he can be safe

  • @Mercury2wo
    @Mercury2wo Před 4 lety +2

    Fantastic.

  • @jonahansen
    @jonahansen Před 2 lety

    Good one!

  • @simonsmith7680
    @simonsmith7680 Před 3 lety +1

    In the US, the police can shoot the thief from the starting position while eating a 🍩

  • @Killua0306
    @Killua0306 Před 4 lety +6

    Actually it is impossible if the thief is smart...The thief can also cross the diagonal as soon as the police crosses ...to maintain the same distance

    • @Noone-vk1cz
      @Noone-vk1cz Před 4 lety +1

      Nope

    • @aps19august
      @aps19august Před 4 lety

      Not possible bcoz the start position will not allow the thief to do so...

    • @jaygreen7494
      @jaygreen7494 Před 4 lety

      even if the thief could get to the diagonal fast enough to cross it before being caught (different start position) he'd still get caught eventually, because the diagonal square is very unsafe for him, allowing the police to control it as soon as he gets there, and guarantee correct parity. You'd need two diagonals setup correctly to allow the thief to never get caught.

    • @jiogcyihsugyiocjfdoivhphvw6821
      @jiogcyihsugyiocjfdoivhphvw6821 Před rokem

      incorrect

  • @christianfunintuscany1147

    The dumb policeman go around the thief with no hope to catch him because he is always one node farther in the pattern. The smart policeman notice that there is a discontinuity in the pattern, and realizes that he can use this to catch the thief. Whatever are the moves of the thief the smart policeman makes sure to pass through the diagonal edge before directing to him. The diagonal is a kind of ‘break’ in the periodicity of the moves and let the policeman to catch the thief wherever he goes in the pattern. After going through the disgonal the policeman can follow the patterns he likes: he has only to make sure the thief do not use the same trick. So we can say that under the given conditions there is always a pattern to the thief.

  • @shaikmajid6554
    @shaikmajid6554 Před 4 lety +1

    Awesome..👏👏👏🤩🤩

  • @jyotishmanhazarika4229

    Easy...so much explanation not required

  • @talkingtal5369
    @talkingtal5369 Před 4 lety

    Very nice puzzle

  • @becalmandquiet881
    @becalmandquiet881 Před 3 lety

    Great video

  • @PabloFreethinker
    @PabloFreethinker Před 5 měsíci

    perfect example of one step forward two steps back.

  • @maruthasalamoorthiviswanat153

    Nice problem sir. This problem like King and Pawn endings in Chess

    • @LOGICALLYYOURS
      @LOGICALLYYOURS  Před 3 lety +1

      Yes Maru.. you're right, the similar tactic is used in triangulation during end games.

  • @zahidhussain251
    @zahidhussain251 Před rokem

    There was a similar question about finding a cat but in this question the cop *knows* all the time where the thief is while in the other puzzle the catcher though getting closer to cat never actually knows or is sure where it is till the last move when he catches it.

  • @Shadow4wolfz
    @Shadow4wolfz Před 4 lety +1

    The idea is there, but this seems like those Math problems where people buy 30 watermelons. It's meant to be put into perspective, but in reality, people would give you weird looks, because life isn't meant to be turn based, thief and cop

  • @anonymousvendetta7902
    @anonymousvendetta7902 Před 4 lety +2

    That thief not smart enough. Beginner I think 🤔

  • @anshum1675
    @anshum1675 Před 4 lety +3

    I just want to point out that caught is spelt incorrectly at 3:28, nice video anyways

  • @no.1ytshorts577
    @no.1ytshorts577 Před 2 lety +1

    I open from google your vidio

  • @diptobasak9546
    @diptobasak9546 Před 3 lety

    best riddle bro best

  • @manikandan-bq3kt
    @manikandan-bq3kt Před 4 lety +1

    Funny video

  • @vijayagrawal557
    @vijayagrawal557 Před 4 lety +1

    Shaandar

  • @harshpandey6725
    @harshpandey6725 Před 3 lety +1

    Lol I just did the last step..

  • @unmeshchougule5666
    @unmeshchougule5666 Před 3 lety

    Awesome

  • @GeorgeT.G.
    @GeorgeT.G. Před rokem

    super riddle

  • @malcolmrodgers295
    @malcolmrodgers295 Před 4 lety +1

    Brilliant

  • @sarithapabbathi7251
    @sarithapabbathi7251 Před 4 lety +1

    Good one and I solved it

  • @sleglik
    @sleglik Před 3 lety

    That riddle is impossible to answer... in Game rules "a person CAN only move" not that they MUST move. If cop will switch node thief will stop move.

  • @iamsatyamsharma
    @iamsatyamsharma Před 4 lety +1

    Amazing

  • @chillmode1881
    @chillmode1881 Před 2 lety

    No, it's impossible. It's impossible if the thief stays on the upper 2nd dot and the thief can stay on the second upper dot even the police uses the move advantage he has. If the police moves up diagonally, the thief can also move right from the second dot, making the moves even again. Therefore, it's impossible. We have to rely on the thief's idiocy to make this possible.

  • @sajithomas1258
    @sajithomas1258 Před 3 lety +1

    What will happen if the thief deciding not to move at same stage to keep 2 step difference with police??

  • @BrianThorstad
    @BrianThorstad Před 3 lety

    Enjoying these puzzles, subscribed

  • @thatoneleaf9895
    @thatoneleaf9895 Před 4 lety +1

    You could make the thief at one end of the diagonal and the cop at the other and then boom

  • @SHUBHAMGUPTA-ly1ir
    @SHUBHAMGUPTA-ly1ir Před 3 lety

    Really logical things that is....a long bow to the channel creator😊

  • @demigod8983
    @demigod8983 Před 4 lety +17

    This concept of 2 segment is not right. The Thief would never be caught like this.

    • @sangitagupta5486
      @sangitagupta5486 Před 4 lety

      Give explanation Jatin Soni

    • @demigod8983
      @demigod8983 Před 4 lety +2

      @@sangitagupta5486 it doesn't matter where the police go. If i am the thief i would not go where he wants me to.

    • @swastiksahu1312
      @swastiksahu1312 Před 4 lety +1

      @@demigod8983 that isnt happening though. You could go anywhere, but you will get trapped if he gets to odd no. Of segments.

    • @nikhilagnihotri1612
      @nikhilagnihotri1612 Před 4 lety

      @@swastiksahu1312 exactly but if the can easily maintain the even no. By moving back and forth at the same place.

    • @snooglers
      @snooglers Před 4 lety +2

      In every iteration, as long as the most logical moves are made, the cop catches the thief. It’s impossible not to.

  • @jgalloantigo6375
    @jgalloantigo6375 Před 4 lety +2

    GG, I took more or less 40 seconds, actually i took too long, this is way easy, in 10 seconds i could've solved it lmao, to tired it's midnight

  • @xxnotmuchxx
    @xxnotmuchxx Před 4 lety +2

    Is there a way where the theif can prevent being caught by moving in other places?

  • @prashantgupta8804
    @prashantgupta8804 Před 4 lety +6

    Wjy would thief come closer to police and take thr bait..

    • @TheDantheman12121
      @TheDantheman12121 Před 4 lety

      The thief does not need to come closer for this to work.

    • @ca352256
      @ca352256 Před 4 lety +1

      Let me simplify. Because both the thief and cop must move together 1 segment per turn, it affects everything. As soon as the cop gets to the bottom right corner, he can then choose to move 1 segment (go up or go left) or move 2 segment (by using the diagonal). With this being said, no matter what move the thief does, all the cop has to do is make sure they are an odd number away. By doing this, the thief will be forced to move and the cop can just follow until the thief is cornered.
      Basically, all I'm saying, is that it doesn't matter what move the thief does. All that matters is what the cop does as he controls the entire situation.

    • @jiogcyihsugyiocjfdoivhphvw6821
      @jiogcyihsugyiocjfdoivhphvw6821 Před rokem

      try again

  • @mythbusterman8541
    @mythbusterman8541 Před 2 měsíci

    Checkers opposition immediately sprang to mind .

  • @ujjwalram3278
    @ujjwalram3278 Před 4 lety +1

    awesome

  • @kurdischertarzan
    @kurdischertarzan Před 4 lety +1

    I Got 10% smarter thank you

  • @V7B817
    @V7B817 Před 4 lety

    It is just one of the case in many , there r other ways in which police can never caught the theif and the answer can be both yes or no,but for this yes is most likely to favour rather than not in this situation, because we r humans and r designed to solve problems.

  • @user-yi1qv1ww8u
    @user-yi1qv1ww8u Před 3 lety

    Is it possible to prevent police catch the thief?
    I know if police reach the diagnose first could catch the thief eventually.
    BUT what if i want to reach the goal which is even if the police come first at diagnose point, but it still could prevent police to catch him, what would it be like?

  • @LuningskiFTW
    @LuningskiFTW Před 4 lety +1

    This is really easy if you’re a chess player

  • @Comedy_baap
    @Comedy_baap Před 2 lety

    3:12 , 3:13 ,3:14 Dude I think out of box like if theif can go right box insted of going left then it can't be possible to catch theif

  • @anshgupta7585
    @anshgupta7585 Před 3 lety +1

    Bro what is the name of the game

  • @bestbabysiblings2891
    @bestbabysiblings2891 Před 3 lety

    Why did the Thief moved nearer to the cop?
    And why did he even move? The rules didn't say that the Thief has had to make a move

  • @kvsalahuddin5
    @kvsalahuddin5 Před 4 lety

    Wow

  • @JalebJay
    @JalebJay Před 3 lety

    I remember seeing this problem on the ARML back in 2012. Third time I took part on a contest where we had to prove everything. Questions on it looked more at least number of cops, or least amount of pay (cops*time), least amount of time, etc.
    One of the problems I remember was to prove the minimum number of cops needed to guarentee to catch a robber given the network was a K_n (complete graph of degree n) network.

  • @rajisampath2094
    @rajisampath2094 Před 4 lety

    The second last step shows the thief coming towards the police 👺

  • @jarredmarasigan4857
    @jarredmarasigan4857 Před 3 lety

    3:12 jesus christ that scared me..

  • @muhammeddilkas9983
    @muhammeddilkas9983 Před 4 lety +1

    Its a robot thief🙄

  • @riffz6065
    @riffz6065 Před 4 lety

    This puzzle was super easy because i'm a chess player. Calculating ahead is what I do.

  • @Alexis-kg1sm
    @Alexis-kg1sm Před 3 lety

    That is for a child...count me in! (the guy who play board games)

  • @andrewrubin4694
    @andrewrubin4694 Před 4 lety +2

    1) Please proof read these puzzles...caugth makes it seem that the thief has a cold.
    2) Obviously, the diagonal line is there for a reason, but moving on it seems to violate one of the original rules because the movement is not on an edge. (This problem can be solved by rewording the puzzle, but the problem exists is the puzzle is phrased)
    3) I have not spent a huge amount of time on this, but if the thief moves more than one space and stays out of the corner, he may be able to stay at large until the officer's next doughnut break

  • @LoneLibraryGaming
    @LoneLibraryGaming Před 3 lety

    Pretty sure I saw this as a main game mechanic somewhere

  • @FoodLoop
    @FoodLoop Před 4 lety

    The bell at odd diagonal was funny as hell

  • @rampanda2361
    @rampanda2361 Před 4 lety +15

    Nonsense logic. The thief had a chance to move to right just after when the police reached at the bottom right corner. But you purpose fully moved the thief to left.

    • @WesselB
      @WesselB Před 4 lety +2

      ram panda If he had moved to the thief to the right, then the police would move via the diagonal and then catch the thief on the next turn.

    • @rshanthikumar6224
      @rshanthikumar6224 Před 4 lety

      But there is no possible for diagonal... Move rigt?? Except in right corner down

    • @rshanthikumar6224
      @rshanthikumar6224 Před 4 lety

      Its nonsense off course..

    • @akasin8202
      @akasin8202 Před 4 lety +1

      @@rshanthikumar6224 the police will still catch the thief

    • @Mihau_desu
      @Mihau_desu Před 4 lety +1

      If at 3:12 thief would have moved to the right, then police officer goes throught diagonal and thief is trapped.
      If he goes left, police officer goes up and catches him
      If thief goes down, police officer goes right and catches him.
      If thief goes up or left... well there's no up nor left.
      However imagine there was. If it would be finite and follow the same rules ( there would be squares without paths throught diagonals ). At some point thief would end up being cornered.
      However if he could just go up forever without bounds, then yeah. He wouldn't ever be cought in most cases.

  • @cww80
    @cww80 Před 3 lety

    Being a chess player helped solving this.

  • @AfridonDeaddark
    @AfridonDeaddark Před 3 lety

    Of course how didn't I get it - police officer run away from thief to catch him easy. So real or Surreal?

  • @desidoodperfect
    @desidoodperfect Před 4 lety +3

    Face .....!?

  • @abdullahalnayem1849
    @abdullahalnayem1849 Před 4 lety +1

    😲 👏❤

  • @snakesvt
    @snakesvt Před 2 lety

    Why would the thief fallow cop as the cop moves right. That doesn’t make sense. You want to distance yourself from the cop not him

  • @kylen6430
    @kylen6430 Před 3 lety

    As a chess player would say...you need to get the opposition

  • @harishankar-cz9tx
    @harishankar-cz9tx Před 3 lety

    you forgot to mention that you must move every turn.

  • @Wattsini91
    @Wattsini91 Před 4 lety +6

    They could just keep going around the first square forever

    • @jiogcyihsugyiocjfdoivhphvw6821
      @jiogcyihsugyiocjfdoivhphvw6821 Před rokem

      ya cause that solves the puzzle.....................................................................................................................................

  • @pratik6707
    @pratik6707 Před 4 lety

    Which software do u use to make the videos??

    • @LOGICALLYYOURS
      @LOGICALLYYOURS  Před 4 lety

      MS PPT

    • @pratik6707
      @pratik6707 Před 4 lety +1

      @@LOGICALLYYOURS 😍thanks, let me try it....
      You are not like others...thanks for telling

  • @balaezthbarriana2303
    @balaezthbarriana2303 Před 4 lety

    at 3:12: Police is bottom right and can choose go diagonal or straight while the thief is on the 4th spot top row... what would happen when the thief did not go left. if he would go right instead.... it would be a even numer of segments or am i wrong?

    • @imoutofnames1
      @imoutofnames1 Před 3 lety +1

      It wouldn't matter. Thief goes right. Police uses diagnial. Thief then needs to go either down or to the left as those are his only remaining options which then puts him directly next to the police just in time for the police next move.

    • @jiogcyihsugyiocjfdoivhphvw6821
      @jiogcyihsugyiocjfdoivhphvw6821 Před rokem

      ur wrong